Minimum operations to find tangent to circle












12












$begingroup$


I've been playing the game Euclidea 3, and I can't really wrap my mind around one of the minimal solutions:



enter image description here



https://www.youtube.com/watch?v=zublg6ZevKo&feature=youtu.be&t=9



The object is to get a tangent line on right of the blue circle with only 3 operations. The minimum solution involves only 2 compass operations (shown above as the gray circles) and the final operation is a straight edge through the point on the blue circle and the intersection of the two compass operations.



My question is: how does this work? How do these two compass operations always result in a point tangential to the blue circle's point?










share|cite|improve this question









$endgroup$












  • $begingroup$
    I thinks we are constructing 2 circles just to get interesting points for blue circle and to join to get tangent to blue circle
    $endgroup$
    – Fawad
    Oct 10 '16 at 2:30
















12












$begingroup$


I've been playing the game Euclidea 3, and I can't really wrap my mind around one of the minimal solutions:



enter image description here



https://www.youtube.com/watch?v=zublg6ZevKo&feature=youtu.be&t=9



The object is to get a tangent line on right of the blue circle with only 3 operations. The minimum solution involves only 2 compass operations (shown above as the gray circles) and the final operation is a straight edge through the point on the blue circle and the intersection of the two compass operations.



My question is: how does this work? How do these two compass operations always result in a point tangential to the blue circle's point?










share|cite|improve this question









$endgroup$












  • $begingroup$
    I thinks we are constructing 2 circles just to get interesting points for blue circle and to join to get tangent to blue circle
    $endgroup$
    – Fawad
    Oct 10 '16 at 2:30














12












12








12





$begingroup$


I've been playing the game Euclidea 3, and I can't really wrap my mind around one of the minimal solutions:



enter image description here



https://www.youtube.com/watch?v=zublg6ZevKo&feature=youtu.be&t=9



The object is to get a tangent line on right of the blue circle with only 3 operations. The minimum solution involves only 2 compass operations (shown above as the gray circles) and the final operation is a straight edge through the point on the blue circle and the intersection of the two compass operations.



My question is: how does this work? How do these two compass operations always result in a point tangential to the blue circle's point?










share|cite|improve this question









$endgroup$




I've been playing the game Euclidea 3, and I can't really wrap my mind around one of the minimal solutions:



enter image description here



https://www.youtube.com/watch?v=zublg6ZevKo&feature=youtu.be&t=9



The object is to get a tangent line on right of the blue circle with only 3 operations. The minimum solution involves only 2 compass operations (shown above as the gray circles) and the final operation is a straight edge through the point on the blue circle and the intersection of the two compass operations.



My question is: how does this work? How do these two compass operations always result in a point tangential to the blue circle's point?







geometry circles geometric-construction tangent-line






share|cite|improve this question













share|cite|improve this question











share|cite|improve this question




share|cite|improve this question










asked Oct 10 '16 at 2:20









GillespieGillespie

163115




163115












  • $begingroup$
    I thinks we are constructing 2 circles just to get interesting points for blue circle and to join to get tangent to blue circle
    $endgroup$
    – Fawad
    Oct 10 '16 at 2:30


















  • $begingroup$
    I thinks we are constructing 2 circles just to get interesting points for blue circle and to join to get tangent to blue circle
    $endgroup$
    – Fawad
    Oct 10 '16 at 2:30
















$begingroup$
I thinks we are constructing 2 circles just to get interesting points for blue circle and to join to get tangent to blue circle
$endgroup$
– Fawad
Oct 10 '16 at 2:30




$begingroup$
I thinks we are constructing 2 circles just to get interesting points for blue circle and to join to get tangent to blue circle
$endgroup$
– Fawad
Oct 10 '16 at 2:30










3 Answers
3






active

oldest

votes


















14












$begingroup$

The construction



In the construction above, we are given the circle centred on $O$ (black) and the point $P$. We pick an arbitrary point $A$ on the circle and construct two more circles (in light grey) which intersect in $B$ and $C$; a segment of the tangent is then $PC$. $Z$ is a point on the black circle, on the other side of the chord $PB$ from $A$, and is solely a component of our proof: that $angle OPC=90^circ$. Line segments of the same length have the same colour (except the ones incident to $Z$).



Say $angle BPA=x$. Since $triangle PAB$ and $triangle CAP$ are congruent and isosceles, $angle APC=x$.



$angle PAB$ is $180^circ-2x$, and since quadrilateral $PABZ$ is cyclic, $angle PAB+angle BZP=180^circ$, or $angle BZP=2x$. By the inscribed angle theorem, $angle BOP=4x$.



Since $triangle BOP$ is isosceles, $angle OPB=frac12(180^circ-4x)=90^circ-2x$. Then
$$angle OPC=angle OPB+angle BPA+angle APC=90^circ-2x+x+x=90^circ$$
Hence $PC$ is tangent to the black circle at $P$.






share|cite|improve this answer









$endgroup$









  • 1




    $begingroup$
    Nice explanation!
    $endgroup$
    – J3soon
    Sep 16 '17 at 20:29



















2












$begingroup$

(1) $angle MXN = 90^0$. (Angles in semi-circle)



(2) $angle RTN = 90^0$. (Line of centers is perpendicular to common chord)



(1) and (2) imply XM // TR.



enter image description here



From (2), we have $triangle RTN cong triangle RTX$. Then, $angle 1 = angle 2$.



Also, $angle 2 = angle 3$ , $angle 2 = angle 4$ and $angle 4 = angle 5$.



Therefore, $angle 5 = angle 1$. This means MRN is tangent to the blue circle by “converse of angles in alternate segment”.






share|cite|improve this answer









$endgroup$





















    1












    $begingroup$

    My version is not a strict formal proof, but rather an intuition behind the idea. I hope it helps to get the point without having to dive into a lot of formal steps and theorems.



    There are actually two major symmetries in this construct that make the whole thing possible.



    The first is the reflection symmetry of the circle p relative to the AB line. It guarantees that the two arcs, DB and BE, are equal, which in turn implies that DE is twice as long as DB.



    Now pretend this construct in motion. When we move point B along the circle p, keeping all given constraints, the DE segment rotates relative to the AD line twice as fast as DB does and, consequently, $angle ADE$ changes twice as fast as $angle ADB$.



    Here we have to note the second major symmetry -- the symmetry of the circles q and s relative to the DB line. This symmetry implies that $angle EDB = angle GDB$.



    Continuing to look at this in motion, we notice that when we move B, say, towards D, the DE segment 'chases' the DB line with twice its speed. At the same time, the DG segment symmetrically 'meets' the DB line with the same speed, but in the opposite direction. This neatly guides us to the notion that the DG segment stays fixed, or, in other words, $angle ADG = const$.



    Now all we're left to do is to find the actual value of $angle ADG$. We can continue to look at the motion described above, pretending that B approaches D. We see that all the lines DE, DB, and DG approach each other and become one line as B becomes D. And this line is clearly tangent to the circle p. But DG was not even moving relative to AD, so it should have been the same tangent line from the very beginning.



    Hope that helps.



    two symmetries relative to blue lines






    share|cite|improve this answer











    $endgroup$













      Your Answer





      StackExchange.ifUsing("editor", function () {
      return StackExchange.using("mathjaxEditing", function () {
      StackExchange.MarkdownEditor.creationCallbacks.add(function (editor, postfix) {
      StackExchange.mathjaxEditing.prepareWmdForMathJax(editor, postfix, [["$", "$"], ["\\(","\\)"]]);
      });
      });
      }, "mathjax-editing");

      StackExchange.ready(function() {
      var channelOptions = {
      tags: "".split(" "),
      id: "69"
      };
      initTagRenderer("".split(" "), "".split(" "), channelOptions);

      StackExchange.using("externalEditor", function() {
      // Have to fire editor after snippets, if snippets enabled
      if (StackExchange.settings.snippets.snippetsEnabled) {
      StackExchange.using("snippets", function() {
      createEditor();
      });
      }
      else {
      createEditor();
      }
      });

      function createEditor() {
      StackExchange.prepareEditor({
      heartbeatType: 'answer',
      autoActivateHeartbeat: false,
      convertImagesToLinks: true,
      noModals: true,
      showLowRepImageUploadWarning: true,
      reputationToPostImages: 10,
      bindNavPrevention: true,
      postfix: "",
      imageUploader: {
      brandingHtml: "Powered by u003ca class="icon-imgur-white" href="https://imgur.com/"u003eu003c/au003e",
      contentPolicyHtml: "User contributions licensed under u003ca href="https://creativecommons.org/licenses/by-sa/3.0/"u003ecc by-sa 3.0 with attribution requiredu003c/au003e u003ca href="https://stackoverflow.com/legal/content-policy"u003e(content policy)u003c/au003e",
      allowUrls: true
      },
      noCode: true, onDemand: true,
      discardSelector: ".discard-answer"
      ,immediatelyShowMarkdownHelp:true
      });


      }
      });














      draft saved

      draft discarded


















      StackExchange.ready(
      function () {
      StackExchange.openid.initPostLogin('.new-post-login', 'https%3a%2f%2fmath.stackexchange.com%2fquestions%2f1961629%2fminimum-operations-to-find-tangent-to-circle%23new-answer', 'question_page');
      }
      );

      Post as a guest















      Required, but never shown

























      3 Answers
      3






      active

      oldest

      votes








      3 Answers
      3






      active

      oldest

      votes









      active

      oldest

      votes






      active

      oldest

      votes









      14












      $begingroup$

      The construction



      In the construction above, we are given the circle centred on $O$ (black) and the point $P$. We pick an arbitrary point $A$ on the circle and construct two more circles (in light grey) which intersect in $B$ and $C$; a segment of the tangent is then $PC$. $Z$ is a point on the black circle, on the other side of the chord $PB$ from $A$, and is solely a component of our proof: that $angle OPC=90^circ$. Line segments of the same length have the same colour (except the ones incident to $Z$).



      Say $angle BPA=x$. Since $triangle PAB$ and $triangle CAP$ are congruent and isosceles, $angle APC=x$.



      $angle PAB$ is $180^circ-2x$, and since quadrilateral $PABZ$ is cyclic, $angle PAB+angle BZP=180^circ$, or $angle BZP=2x$. By the inscribed angle theorem, $angle BOP=4x$.



      Since $triangle BOP$ is isosceles, $angle OPB=frac12(180^circ-4x)=90^circ-2x$. Then
      $$angle OPC=angle OPB+angle BPA+angle APC=90^circ-2x+x+x=90^circ$$
      Hence $PC$ is tangent to the black circle at $P$.






      share|cite|improve this answer









      $endgroup$









      • 1




        $begingroup$
        Nice explanation!
        $endgroup$
        – J3soon
        Sep 16 '17 at 20:29
















      14












      $begingroup$

      The construction



      In the construction above, we are given the circle centred on $O$ (black) and the point $P$. We pick an arbitrary point $A$ on the circle and construct two more circles (in light grey) which intersect in $B$ and $C$; a segment of the tangent is then $PC$. $Z$ is a point on the black circle, on the other side of the chord $PB$ from $A$, and is solely a component of our proof: that $angle OPC=90^circ$. Line segments of the same length have the same colour (except the ones incident to $Z$).



      Say $angle BPA=x$. Since $triangle PAB$ and $triangle CAP$ are congruent and isosceles, $angle APC=x$.



      $angle PAB$ is $180^circ-2x$, and since quadrilateral $PABZ$ is cyclic, $angle PAB+angle BZP=180^circ$, or $angle BZP=2x$. By the inscribed angle theorem, $angle BOP=4x$.



      Since $triangle BOP$ is isosceles, $angle OPB=frac12(180^circ-4x)=90^circ-2x$. Then
      $$angle OPC=angle OPB+angle BPA+angle APC=90^circ-2x+x+x=90^circ$$
      Hence $PC$ is tangent to the black circle at $P$.






      share|cite|improve this answer









      $endgroup$









      • 1




        $begingroup$
        Nice explanation!
        $endgroup$
        – J3soon
        Sep 16 '17 at 20:29














      14












      14








      14





      $begingroup$

      The construction



      In the construction above, we are given the circle centred on $O$ (black) and the point $P$. We pick an arbitrary point $A$ on the circle and construct two more circles (in light grey) which intersect in $B$ and $C$; a segment of the tangent is then $PC$. $Z$ is a point on the black circle, on the other side of the chord $PB$ from $A$, and is solely a component of our proof: that $angle OPC=90^circ$. Line segments of the same length have the same colour (except the ones incident to $Z$).



      Say $angle BPA=x$. Since $triangle PAB$ and $triangle CAP$ are congruent and isosceles, $angle APC=x$.



      $angle PAB$ is $180^circ-2x$, and since quadrilateral $PABZ$ is cyclic, $angle PAB+angle BZP=180^circ$, or $angle BZP=2x$. By the inscribed angle theorem, $angle BOP=4x$.



      Since $triangle BOP$ is isosceles, $angle OPB=frac12(180^circ-4x)=90^circ-2x$. Then
      $$angle OPC=angle OPB+angle BPA+angle APC=90^circ-2x+x+x=90^circ$$
      Hence $PC$ is tangent to the black circle at $P$.






      share|cite|improve this answer









      $endgroup$



      The construction



      In the construction above, we are given the circle centred on $O$ (black) and the point $P$. We pick an arbitrary point $A$ on the circle and construct two more circles (in light grey) which intersect in $B$ and $C$; a segment of the tangent is then $PC$. $Z$ is a point on the black circle, on the other side of the chord $PB$ from $A$, and is solely a component of our proof: that $angle OPC=90^circ$. Line segments of the same length have the same colour (except the ones incident to $Z$).



      Say $angle BPA=x$. Since $triangle PAB$ and $triangle CAP$ are congruent and isosceles, $angle APC=x$.



      $angle PAB$ is $180^circ-2x$, and since quadrilateral $PABZ$ is cyclic, $angle PAB+angle BZP=180^circ$, or $angle BZP=2x$. By the inscribed angle theorem, $angle BOP=4x$.



      Since $triangle BOP$ is isosceles, $angle OPB=frac12(180^circ-4x)=90^circ-2x$. Then
      $$angle OPC=angle OPB+angle BPA+angle APC=90^circ-2x+x+x=90^circ$$
      Hence $PC$ is tangent to the black circle at $P$.







      share|cite|improve this answer












      share|cite|improve this answer



      share|cite|improve this answer










      answered Oct 10 '16 at 4:23









      Parcly TaxelParcly Taxel

      44.7k1376110




      44.7k1376110








      • 1




        $begingroup$
        Nice explanation!
        $endgroup$
        – J3soon
        Sep 16 '17 at 20:29














      • 1




        $begingroup$
        Nice explanation!
        $endgroup$
        – J3soon
        Sep 16 '17 at 20:29








      1




      1




      $begingroup$
      Nice explanation!
      $endgroup$
      – J3soon
      Sep 16 '17 at 20:29




      $begingroup$
      Nice explanation!
      $endgroup$
      – J3soon
      Sep 16 '17 at 20:29











      2












      $begingroup$

      (1) $angle MXN = 90^0$. (Angles in semi-circle)



      (2) $angle RTN = 90^0$. (Line of centers is perpendicular to common chord)



      (1) and (2) imply XM // TR.



      enter image description here



      From (2), we have $triangle RTN cong triangle RTX$. Then, $angle 1 = angle 2$.



      Also, $angle 2 = angle 3$ , $angle 2 = angle 4$ and $angle 4 = angle 5$.



      Therefore, $angle 5 = angle 1$. This means MRN is tangent to the blue circle by “converse of angles in alternate segment”.






      share|cite|improve this answer









      $endgroup$


















        2












        $begingroup$

        (1) $angle MXN = 90^0$. (Angles in semi-circle)



        (2) $angle RTN = 90^0$. (Line of centers is perpendicular to common chord)



        (1) and (2) imply XM // TR.



        enter image description here



        From (2), we have $triangle RTN cong triangle RTX$. Then, $angle 1 = angle 2$.



        Also, $angle 2 = angle 3$ , $angle 2 = angle 4$ and $angle 4 = angle 5$.



        Therefore, $angle 5 = angle 1$. This means MRN is tangent to the blue circle by “converse of angles in alternate segment”.






        share|cite|improve this answer









        $endgroup$
















          2












          2








          2





          $begingroup$

          (1) $angle MXN = 90^0$. (Angles in semi-circle)



          (2) $angle RTN = 90^0$. (Line of centers is perpendicular to common chord)



          (1) and (2) imply XM // TR.



          enter image description here



          From (2), we have $triangle RTN cong triangle RTX$. Then, $angle 1 = angle 2$.



          Also, $angle 2 = angle 3$ , $angle 2 = angle 4$ and $angle 4 = angle 5$.



          Therefore, $angle 5 = angle 1$. This means MRN is tangent to the blue circle by “converse of angles in alternate segment”.






          share|cite|improve this answer









          $endgroup$



          (1) $angle MXN = 90^0$. (Angles in semi-circle)



          (2) $angle RTN = 90^0$. (Line of centers is perpendicular to common chord)



          (1) and (2) imply XM // TR.



          enter image description here



          From (2), we have $triangle RTN cong triangle RTX$. Then, $angle 1 = angle 2$.



          Also, $angle 2 = angle 3$ , $angle 2 = angle 4$ and $angle 4 = angle 5$.



          Therefore, $angle 5 = angle 1$. This means MRN is tangent to the blue circle by “converse of angles in alternate segment”.







          share|cite|improve this answer












          share|cite|improve this answer



          share|cite|improve this answer










          answered Oct 10 '16 at 5:33









          MickMick

          11.9k21641




          11.9k21641























              1












              $begingroup$

              My version is not a strict formal proof, but rather an intuition behind the idea. I hope it helps to get the point without having to dive into a lot of formal steps and theorems.



              There are actually two major symmetries in this construct that make the whole thing possible.



              The first is the reflection symmetry of the circle p relative to the AB line. It guarantees that the two arcs, DB and BE, are equal, which in turn implies that DE is twice as long as DB.



              Now pretend this construct in motion. When we move point B along the circle p, keeping all given constraints, the DE segment rotates relative to the AD line twice as fast as DB does and, consequently, $angle ADE$ changes twice as fast as $angle ADB$.



              Here we have to note the second major symmetry -- the symmetry of the circles q and s relative to the DB line. This symmetry implies that $angle EDB = angle GDB$.



              Continuing to look at this in motion, we notice that when we move B, say, towards D, the DE segment 'chases' the DB line with twice its speed. At the same time, the DG segment symmetrically 'meets' the DB line with the same speed, but in the opposite direction. This neatly guides us to the notion that the DG segment stays fixed, or, in other words, $angle ADG = const$.



              Now all we're left to do is to find the actual value of $angle ADG$. We can continue to look at the motion described above, pretending that B approaches D. We see that all the lines DE, DB, and DG approach each other and become one line as B becomes D. And this line is clearly tangent to the circle p. But DG was not even moving relative to AD, so it should have been the same tangent line from the very beginning.



              Hope that helps.



              two symmetries relative to blue lines






              share|cite|improve this answer











              $endgroup$


















                1












                $begingroup$

                My version is not a strict formal proof, but rather an intuition behind the idea. I hope it helps to get the point without having to dive into a lot of formal steps and theorems.



                There are actually two major symmetries in this construct that make the whole thing possible.



                The first is the reflection symmetry of the circle p relative to the AB line. It guarantees that the two arcs, DB and BE, are equal, which in turn implies that DE is twice as long as DB.



                Now pretend this construct in motion. When we move point B along the circle p, keeping all given constraints, the DE segment rotates relative to the AD line twice as fast as DB does and, consequently, $angle ADE$ changes twice as fast as $angle ADB$.



                Here we have to note the second major symmetry -- the symmetry of the circles q and s relative to the DB line. This symmetry implies that $angle EDB = angle GDB$.



                Continuing to look at this in motion, we notice that when we move B, say, towards D, the DE segment 'chases' the DB line with twice its speed. At the same time, the DG segment symmetrically 'meets' the DB line with the same speed, but in the opposite direction. This neatly guides us to the notion that the DG segment stays fixed, or, in other words, $angle ADG = const$.



                Now all we're left to do is to find the actual value of $angle ADG$. We can continue to look at the motion described above, pretending that B approaches D. We see that all the lines DE, DB, and DG approach each other and become one line as B becomes D. And this line is clearly tangent to the circle p. But DG was not even moving relative to AD, so it should have been the same tangent line from the very beginning.



                Hope that helps.



                two symmetries relative to blue lines






                share|cite|improve this answer











                $endgroup$
















                  1












                  1








                  1





                  $begingroup$

                  My version is not a strict formal proof, but rather an intuition behind the idea. I hope it helps to get the point without having to dive into a lot of formal steps and theorems.



                  There are actually two major symmetries in this construct that make the whole thing possible.



                  The first is the reflection symmetry of the circle p relative to the AB line. It guarantees that the two arcs, DB and BE, are equal, which in turn implies that DE is twice as long as DB.



                  Now pretend this construct in motion. When we move point B along the circle p, keeping all given constraints, the DE segment rotates relative to the AD line twice as fast as DB does and, consequently, $angle ADE$ changes twice as fast as $angle ADB$.



                  Here we have to note the second major symmetry -- the symmetry of the circles q and s relative to the DB line. This symmetry implies that $angle EDB = angle GDB$.



                  Continuing to look at this in motion, we notice that when we move B, say, towards D, the DE segment 'chases' the DB line with twice its speed. At the same time, the DG segment symmetrically 'meets' the DB line with the same speed, but in the opposite direction. This neatly guides us to the notion that the DG segment stays fixed, or, in other words, $angle ADG = const$.



                  Now all we're left to do is to find the actual value of $angle ADG$. We can continue to look at the motion described above, pretending that B approaches D. We see that all the lines DE, DB, and DG approach each other and become one line as B becomes D. And this line is clearly tangent to the circle p. But DG was not even moving relative to AD, so it should have been the same tangent line from the very beginning.



                  Hope that helps.



                  two symmetries relative to blue lines






                  share|cite|improve this answer











                  $endgroup$



                  My version is not a strict formal proof, but rather an intuition behind the idea. I hope it helps to get the point without having to dive into a lot of formal steps and theorems.



                  There are actually two major symmetries in this construct that make the whole thing possible.



                  The first is the reflection symmetry of the circle p relative to the AB line. It guarantees that the two arcs, DB and BE, are equal, which in turn implies that DE is twice as long as DB.



                  Now pretend this construct in motion. When we move point B along the circle p, keeping all given constraints, the DE segment rotates relative to the AD line twice as fast as DB does and, consequently, $angle ADE$ changes twice as fast as $angle ADB$.



                  Here we have to note the second major symmetry -- the symmetry of the circles q and s relative to the DB line. This symmetry implies that $angle EDB = angle GDB$.



                  Continuing to look at this in motion, we notice that when we move B, say, towards D, the DE segment 'chases' the DB line with twice its speed. At the same time, the DG segment symmetrically 'meets' the DB line with the same speed, but in the opposite direction. This neatly guides us to the notion that the DG segment stays fixed, or, in other words, $angle ADG = const$.



                  Now all we're left to do is to find the actual value of $angle ADG$. We can continue to look at the motion described above, pretending that B approaches D. We see that all the lines DE, DB, and DG approach each other and become one line as B becomes D. And this line is clearly tangent to the circle p. But DG was not even moving relative to AD, so it should have been the same tangent line from the very beginning.



                  Hope that helps.



                  two symmetries relative to blue lines







                  share|cite|improve this answer














                  share|cite|improve this answer



                  share|cite|improve this answer








                  edited Jun 8 '18 at 0:39

























                  answered Jun 8 '18 at 0:25









                  alkolnalkoln

                  1112




                  1112






























                      draft saved

                      draft discarded




















































                      Thanks for contributing an answer to Mathematics Stack Exchange!


                      • Please be sure to answer the question. Provide details and share your research!

                      But avoid



                      • Asking for help, clarification, or responding to other answers.

                      • Making statements based on opinion; back them up with references or personal experience.


                      Use MathJax to format equations. MathJax reference.


                      To learn more, see our tips on writing great answers.




                      draft saved


                      draft discarded














                      StackExchange.ready(
                      function () {
                      StackExchange.openid.initPostLogin('.new-post-login', 'https%3a%2f%2fmath.stackexchange.com%2fquestions%2f1961629%2fminimum-operations-to-find-tangent-to-circle%23new-answer', 'question_page');
                      }
                      );

                      Post as a guest















                      Required, but never shown





















































                      Required, but never shown














                      Required, but never shown












                      Required, but never shown







                      Required, but never shown

































                      Required, but never shown














                      Required, but never shown












                      Required, but never shown







                      Required, but never shown







                      Popular posts from this blog

                      Human spaceflight

                      Can not write log (Is /dev/pts mounted?) - openpty in Ubuntu-on-Windows?

                      File:DeusFollowingSea.jpg